LSAT and Law School Admissions Forum

Get expert LSAT preparation and law school admissions advice from PowerScore Test Preparation.

 Administrator
PowerScore Staff
  • PowerScore Staff
  • Posts: 8917
  • Joined: Feb 02, 2011
|
#33104
Complete Question Explanation

Assumption. The correct answer choice is (B)

The author compares two advertisements for a TV drama: the network’s ad, and the ad favored by the program’s producers. The network’s ad apparently misrepresents the program, which the author fears would create false expectations and lower the chances that viewers would return to watch subsequent episodes. As a result, the author believes that the ad favored by the program’s producers would be more effective at attracting loyal viewers than the network’s ad.

The stimulus has a confusing structure, with the conclusion appearing in the beginning of the second sentence (notice the conclusion indicator “thus”). The first sentence introduces a premise, as does the second clause of the second sentence. Note that semicolons after a conclusionary claim generally introduce evidence for that claim, acting as premise indicators. The argument, when restructured, can be summarized as follows:
  • Premise: ..... The TV network’s ad for its new drama misrepresents what the drama is like.

    Premise: ..... Viewers who are misled about the first episode are unlikely to continue watching the program.

    Cause ..... ..... ..... ..... ..... Effect/Cause ..... ..... ..... ..... Effect

    Network’s ad ..... :arrow: ..... Misrepresents drama ..... :arrow: ..... Unlikely to continue watching

    Conclusion: ..... The ad preferred by the producers would be more effective in attracting loyal viewers than the ..... ..... ..... ..... network’s ad.
Despite the heavy dose of causal relationships that underlie this argument, its main flaw has little to do with causal reasoning. This is because the conclusion does not attempt to infer a causal relationship or presume one to exist; it merely relies on such relationships as matters of fact. The main problem involves a Relativity Flaw: the premises only establish that the network’s ad will be ineffective, not that it would be less effective than the ad favored by the program’s producers. The latter is a “rogue” element in this argument, as the author provides no information to help us compare the merits of both ads. Naturally, we are left to wonder what makes one of them any more effective than the other.

Test makers often play on the distinction between relative states and absolute states within arguments. This distinction is often subtle and can be difficult to spot. Because this is an assumption question, the answer you select must contain a statement upon which the argument depends, i.e. a statement that is necessary for the conclusion to be true. Typically, if you see a new or “rogue” element in the conclusion, look for a Supporter assumption answer that links the new element (the ad preferred by the producers) back to the premises.

Answer choice (A): This answer choice contains an exaggeration. If the author worries that viewers would be misled by the network’s advertisement, clearly she must assume that at least some viewers tune in to the first episode because of the ad. We need not assume, however, that most viewers do so.

Answer choice (B): This is the correct answer choice. Since the author never explicitly described the merits of the ad favored by the program’s producers, a correct Supporter Assumption needs to establish that this ad would not have misrepresented the program. Answer choice (B) fits the bill. To verify your answer, apply the Assumption Negation Technique and ask yourself, “What would the author say to this negation?”
  • “The advertisement that the program’s producers favored would have grossly misrepresented what the program would be like.”
The logical opposite of answer choice (B) implies that both ads would have been just as ineffective, weakening the conclusion of the argument. Therefore, answer choice (B) contains an assumption necessary for the conclusion to be true.

Answer choice (C): As with answer choice (A), this one contains an exaggeration (“most”). Granted, if the network’s ad is as ineffective as the author claims, then many of those who become loyal viewers of the drama will probably do so in spite of having seen it, not because of it. This may be an implication of the author’s conclusion, but it is certainly not an assumption upon which it depends. Furthermore, no assumptions have been made with respect to the behavior of “most” people.

Answer choice (D): As with answer choices (A) and (C), this one contains an exaggeration (“almost all”). The author assumes that the ad favored by the program’s producers would turn more first-time viewers into loyal followers than the network’s ad would, not that it would turn most of them into loyal followers. If the latter were true, then the conclusion would clearly be strengthened. Such a statement, however, is not necessary for the conclusion to be true, because even if less than half of the first-time viewers return to watch the show as a result of the ad favored by the show’s producers, it is still possible that this ad is more effective than the rejected alternative.

Answer choice (E): As with answer choices (A), (C), and (D), the author makes no assumptions with respect to the behavior of “most” people. The argument is only concerned with whether first-time viewers who watch the first episode of the show would return to watch subsequent episodes. Those who become loyal viewers without seeing the first episode represent a group that is irrelevant to the author’s conclusion.
 actionjackson
  • Posts: 22
  • Joined: Nov 22, 2016
|
#32656
For this Assumption question, I had narrowed my choices down to B and D. I believe noticing new info (the producer's favored ad) in the conclusion is what led me to narrow down my choices to B and D but, having incorrectly chosen answer choice D for this question, I'm wondering why that answer choice is incorrect and answer choice B is correct. I had identified the conclusion of this argument as "it (the ad) will not as effectively attract the sort of viewers likely to continue watching as would the ad that the producers favored". Because of premises: "The network's ad grossly misrepresents what the program is like" and "People who tune in to the first episode based on false expectations will be unlikely to watch subsequent episodes". Is D incorrect because it is simply something that could be true but doesn't have to be true for the argument to stand? Assumption questions (and to an extent weakeners) always seem to be my short coming on the LR section.
 Adam Tyson
PowerScore Staff
  • PowerScore Staff
  • Posts: 5153
  • Joined: Apr 14, 2011
|
#32664
You've got it, I think, actionjackson. D isn't a necessary assumption of the argument, although it would certainly help to strengthen it.

Negating "almost all" is tricky. Maybe "Some amount that is less than almost all"? The reason this negation doesn't ruin the argument is because we don't need the producer's preferred ad to be tremendously successful; we only need it to be more successful than the network's chosen ad. Maybe 2% of viewers will stay if they watched based on the network's ad, but 15% would have stayed if they watched based on the producer's preferred ad? That's all that our conclusion requires.

Good work!
User avatar
 Jonathan Evans
PowerScore Staff
  • PowerScore Staff
  • Posts: 726
  • Joined: Jun 09, 2016
|
#32665
Hi, Jackson,

Great question about a fascinating assumption problem, one which presents multiple challenges, both in the analysis of the stimulus and in the wording of the answer choices. First off, great job identifying the conclusion. Even though it's sandwiched in the middle of the stimulus, the indicator word "thus" and the semicolon introducing further support give it away.

Now let's zero in on the crux of your question, which is about your difficulty with assumption questions and why answer choice D is incorrect. Let's tackle each of these points separately. First, to review, assumption questions instruct you to find a necessary precondition, a required but unstated belief the author must hold for his conclusion to be valid.

I sometimes like to explain this concept as a bare-minimum requirement, the thing that absolutely has to be true for the conclusion to make sense. This is why the Assumption Negation Test ™ works: logically negate a required assumption and the conclusion becomes farcical, stupid, nonsensical, or impossible.

With this established, let's examine your two answer choices. Consider D first. Always remind yourself of the conclusion: this guy thinks the producers' ad would be more successful than the one the network aired. Now ask yourself, is it absolutely necessary that if they had opted for the producers' ad, almost everyone who saw it would have tuned in for the second episode? While this information might certainly strengthen the conclusion, perhaps we can agree it need not be "almost everyone." What if it were just "most people" or even "a few more viewers than it would have been with the network ad?" Either one of these situations would be consistent with the conclusion but not necessarily consistent with this answer choice. This is how the Assumption Negation Test™ works. Imagine not almost everyone who saw the producers' ad tuned in for the second episode; could the conclusion about the relative success of the show still be possible? Sure, why not.

Now run answer choice B by this test. What if the producers' ad had done exactly the same thing? What if the producers' ad had also grossly misrepresented the show? Would the conclusion make any sense at all? No way! This is a necessary assumption. I hope this explanation helps.
 NeverMissing
  • Posts: 35
  • Joined: Feb 21, 2017
|
#34150
I am having trouble seeing why B is an assumption on which the argument relies. The language seems too strong in this instance, overstepping the premises laid out in the stimulus.

The conclusion of the argument is (paraphrased): The network's ad will not attract viewers as effectively as the producers' ad. As I understand it, this means that all that is needed to validate this conclusion is evidence that the producer's ad would attract at least one more person to become a loyal viewer than the network's ad. It could possibly attract many more people, but it is only required to attract one more person to be more effective than the network's ad, and thus validate the conclusion.

I don't see how B—The producers' ad wouldn't have grossly misrepresented what the program is like—is a necessary assumption. Isn't all that is needed to validate this conclusion the premise that the producer's ad will grossly misrepresent what the program is like to a lesser extent than the network's ad? It could not misrepresent the program at all, sure, but I don't see why that is required. Because all the conclusion is arguing is that the producers' ad is more effective than the network's ad—not that it is effective in general. It could still be massively ineffective but just barely eke out more effectiveness than the network's ad. And an ad's effectiveness is measured by the extent to which it grossly misrepresents what the program is like. So couldn't the producers' ad still grossly misrepresent to some degree and still be more effective?

Thanks for your help on this one!
 Kristina Moen
PowerScore Staff
  • PowerScore Staff
  • Posts: 230
  • Joined: Nov 17, 2016
|
#34189
Hi NeverMissing,

First, I want to make sure that we are using the same language. The correct answer choice on an Assumption question is something that is necessary for the conclusion to be true, not something that will validate or justify the conclusion. And remember, it is something that is necessary for the argument in front of you.. Here, the author gives us two premises for his conclusion:
1. The television network’s advertisement for its new medical drama grossly misrepresents what that program is like.
2. People who tune in to the first episode based on false expectations will be unlikely to watch subsequent episodes.

You are correct to pick up on the conclusion being a statement of relative effectiveness. That is good, and you should always pay attention to whether statements are absolute in nature or relative in nature. That said, here the phrase "grossly misrepresents" is used in both answer choice (B) and the premise. There is no reason to believe that "grossly misrepresents" is a matter of degree. Further, the 2nd premise tells us that people who tune in with false expectations are unlikely to watch subsequent episodes - again, there's no matter of degree here. Expectations are false or they are not. Thus, an assumption of this argument is that the program producer's ads would have not have created false expectations.

Let me give you an analogous argument: "Sam makes spicy food. Thus, children will be less likely to eat Sam's food than Anna's food because children do not like spicy food." The assumption here is that is that Anna does not make spicy food. You may see another argument like this on the test, so be aware that when comparing two people or things (as you picked up!) in the conclusion when the premise only tells you about one person/thing means that the argument makes some assumptions (either that they are similar or different).
 ssnasir
  • Posts: 22
  • Joined: Feb 22, 2020
|
#74361
Hi there,
I got this question right but I was wondering that to negate D I converted the "almost everyone" to some i.e. if they aired the producers' add then some of the viewers who tuned into the first episode would tune into subsequent episodes as well" which for me didn't seem hurt the argument. I usually come into problem with the logical opposite/polar opposites thing.
I just wanted to see if that is an appropriate approach?
Thank you,
Noor
 Jeremy Press
PowerScore Staff
  • PowerScore Staff
  • Posts: 1000
  • Joined: Jun 12, 2017
|
#74376
Hi Noor,

Technically, "some" is only the logical opposite of "none." Since "some" can include both "almost all" and "all," it's not a good term to use to negate the term "almost all" in answer choice D.

The better negated form of answer choice D is this (changes in bold): "If the advertisement that the program’s producers favored were used instead of the network’s advertisement, then not almost all of the viewers who tuned in to the first episode would tune in to subsequent episodes as well." What does "not almost all" mean? It means something less than almost all. As you note, though, this negation does not damage the argument or the conclusion, so answer choice D is not truly necessary.

And by the way, if you're ever having trouble negating a term, the best way to do it is to tack a "not" to the beginning of the term (or remove a not, if it's already there!).

I hope this helps!

Jeremy
 blade21cn
  • Posts: 100
  • Joined: May 21, 2019
|
#84193
I eliminated (D) because it is a mistaken negation of the stimulus, which would not help out the argument. We only know based upon the stimulus what will happen if the television network's advertisement is used instead of the advertisement that the program's producers favored. We do not know what will happen if the advertisement that the program's producers favored were used instead. So any prediction will not be supported. Although people may argue the producers' favored advertisement must be more effective, we still don't know how that effectiveness would manifest itself. Also, such a line of thinking is tantamount to circular reasoning. Is this rationale valid thinking?

Lastly, I analyzed the stimulus differently. The second/last sentence starts with "thus," which indicates that the whole sentence is the conclusion. My understanding of the use of semicolon is to join two independent ideas that are given equal rank - similar to the use of the conjunction "and." Moreover, the second clause in the second/last sentence contains judgment/prediction/opinion language - "will be unlikely to ...," which is conclusion material and would require support. So my interpretation is that this stimulus contains one premise with two conclusions. Any thoughts? Thanks!
 Adam Tyson
PowerScore Staff
  • PowerScore Staff
  • Posts: 5153
  • Joined: Apr 14, 2011
|
#84731
The second/last sentence starts with "thus," which indicates that the whole sentence is the conclusion.
This is not correct, blade21cn, and this sort of analysis could land you in some trouble on future questions. A single sentence that starts with a conclusion indicator can still contain both premises and conclusions, whether the phrases are joined by a comma or by a semicolon. The last sentence is both a premise about people with false expectations, and a conclusion about the ad being less effective than the alternative.

I would not call answer D a Mistaken Negation. Since the assumption is something like "if the other ad had been used it would not have given as much of a false impression", a Mistaken Negation would be more like "if the other ad was not used if would have given at least as much of a false impression." If that was an answer choice it would probably have caused a bunch of confusion, but it would still have been incorrect.

The real problem with answer D is that it is too strong, in that the author doesn't have to assume that almost all people would tune in again. Instead, the author only has to assume that fewer people would get a false impression based on that other ad.

And just to clarify, we aren't making a prediction when we select answer B. We are just selecting what the author must have believed if their argument is to be accepted as valid. We don't have to agree that it's true, and we don't have to believe the assumption is valid; we just need to understand what the author must have believed was true.

Get the most out of your LSAT Prep Plus subscription.

Analyze and track your performance with our Testing and Analytics Package.